Đến nội dung

Hình ảnh

Chứng minh rằng : $A< \frac{1}{\sqrt{3n+1}}$ với $A$ cho trước.

* - - - - 1 Bình chọn

  • Please log in to reply
Chủ đề này có 2 trả lời

#1
hoangquan9x

hoangquan9x

    Hạ sĩ

  • Pre-Member
  • 67 Bài viết

 

 Cho A=$\frac{1}{2}$.$\frac{3}{4}\cdot \frac{5}{6}\cdot ...\cdot \frac{2n-1}{2n}$ $\left ( n\in N,n\geq 2 \right )$

          Chứng minh rằng :

                  a,  $A< \frac{1}{\sqrt{3n+1}}$

                  b,$A< \frac{1}{\sqrt{2n+1}}$

 


Bài viết đã được chỉnh sửa nội dung bởi sieusieu90: 20-08-2014 - 21:42


#2
chardhdmovies

chardhdmovies

    Thiếu úy

  • Thành viên
  • 638 Bài viết

mấy bài này đơn giản là dùng qui nạp,nhưng anh sẽ làm theo 1 cách khác

$a)$

có $A=\frac{1}{2}.\frac{3}{4}...\frac{2n-1}{2n}$

mà $A<\frac{2}{3}.\frac{4}{5}...\frac{2n}{2n+1}$

$\Rightarrow A^2<\frac{1.3...(2n-1)}{2.4...(2n)}.\frac{2.4...(2n)}{3.5...(2n+1)}=\frac{1}{2n+1}\Rightarrow A<\frac{1}{\sqrt{2n+1}}$

$b)$ta chỉ cần chứng minh $\frac{2k-1}{2k}<\frac{\sqrt{3k-2}}{\sqrt{3k+1}}$

cái này bình phương lên là được rồi từ đó mà áp dụng


                                                                                    chúng tôi là 3 người từ lớp 10 cá tính:NRC,NTP,A-Q


#3
hoangquan9x

hoangquan9x

    Hạ sĩ

  • Pre-Member
  • 67 Bài viết

Cảm ơn anh nghe ,em không lắm dõ cách quy nạp .Anh chị nào chỉ em luôn nhe.






0 người đang xem chủ đề

0 thành viên, 0 khách, 0 thành viên ẩn danh